LSAT and Law School Admissions Forum

Get expert LSAT preparation and law school admissions advice from PowerScore Test Preparation.

 Administrator
PowerScore Staff
  • PowerScore Staff
  • Posts: 8919
  • Joined: Feb 02, 2011
|
#61031
Please post your questions below!
User avatar
 adricristofari
  • Posts: 3
  • Joined: Oct 06, 2022
|
#97993
why can't J go third? I was stuck between fourth and fifth and couldn't figure out how those work
 Rachael Wilkenfeld
PowerScore Staff
  • PowerScore Staff
  • Posts: 1358
  • Joined: Dec 15, 2011
|
#98010
Hi adricristofari,

Sometimes, you'll get a question like this---a global question about an inference you just did not see upfront. There are a few options. First, you can skip the question until you do more of the game. Until you see more questions, more answers. Question 9, for example, gives us a situation where J is 5, which eliminates answer choice (E) for number 8. But let's say you want to look at this without referencing other questions. Here's how we'd do that:

Let's think about why J couldn't go somewhere. J is only in one rule. J cannot be immediately before or after V.

Our next step would be to think about what would force J to be next to V. V also has some limitations. It has to go before K and S, which means it cannot go 5 or 6.

If J is 1, there are no problems. J would be very far from V because RN would be in between J and V. R and N also provide a buffer when J is in 2.

But what happens when J is in 3? R would be 1, N would be 2. J would be 3. Then we'd only have slot 4 for V to go before K and S. That puts J right before V which is the one thing that cannot happen. That's why answer choice (C) is our correct answer.

Just to be complete, let's try J in 4 and 5. If J is in 4, we can have the order VRNJKS. V and J can be separated by RN again.
If J is in 5, we could have VRNKJS. That's also fine. RNK all separate V from J here.

The key to this question is to notice the interaction between the VJ/JV not block and the R/N relationship. And once you solve this question, you can add the J not rule to slot 3.

Hope that helps!

Get the most out of your LSAT Prep Plus subscription.

Analyze and track your performance with our Testing and Analytics Package.